32 svar
197 visningar
Arup 1124
Postad: 10 jul 09:07

Avgör om gränsvärdet existerar ?

Har lite problem med denna 

"Avgör om följande gränsvärde existerar 

limxx2x+1-x2x+2 

Arup skrev:

Har lite problem med denna 

"Avgör om följande gränsvärde existerar 

limxx2x+1-x2x+2 

Har du problem med att förstå vad det är man vill att du skall göra, eller problem med att göra det?

Mitt första försök skulle vara att skriva uttrycket som ETT bråk.

Arup 1124
Postad: 10 jul 09:12

Problem med vad man ska göra

Då fick du ett tips i mitt första inlägg. Hur ser uttrycket ut när du har gjort så?

Arup 1124
Postad: 10 jul 15:06
Smaragdalena skrev:

Då fick du ett tips i mitt första inlägg. Hur ser uttrycket ut när du har gjort så?

Så här:

Du tappade bort lim vid sista implikationspilen. Bryt ut x2 både i täljaren och nämnaren och låt x gå mot oändligheten. Vad händer?

shkan 215
Postad: 10 jul 16:30 Redigerad: 10 jul 16:43
Smaragdalena skrev:

Du tappade bort lim vid sista implikationspilen. Bryt ut x2 både i täljaren och nämnaren och låt x gå mot oändligheten. Vad händer?

Behöver man inte göra detta med L'hopitals regel? Dessutom två gånger för att få att gränsvärdet blir 1?

 

Edit: Jag inser vad du menar nu. Bra lösningsmetod!

naytte 4385 – Moderator
Postad: 10 jul 17:06 Redigerad: 10 jul 17:07

I fallet med rationella funktioner behöver man aldrig regeln. Dessutom ska man vara försiktig med L'Hôpital, det kan bli logiskt fel väldigt fort. Exempelvis kan man inte använda L'Hôpital i det här gränsvärdet:

limx0sinxx\displaystyle \lim_{x \to 0} \frac{\sin x}{x}

Kan du se varför?


Tillägg: 10 jul 2024 17:06

Oj, såg inte att detta inte var shkans tråd. Om du inte ser varför får du gärna skapa en ny tråd, @shkan! :)

shkan 215
Postad: 10 jul 17:52 Redigerad: 10 jul 17:58
naytte skrev:

I fallet med rationella funktioner behöver man aldrig regeln. Dessutom ska man vara försiktig med L'Hôpital, det kan bli logiskt fel väldigt fort. Exempelvis kan man inte använda L'Hôpital i det här gränsvärdet:

limx0sinxx\displaystyle \lim_{x \to 0} \frac{\sin x}{x}

Kan du se varför?


Tillägg: 10 jul 2024 17:06

Oj, såg inte att detta inte var shkans tråd. Om du inte ser varför får du gärna skapa en ny tråd, @shkan! :)

Man får inte använda sig av L'hopitals regel i det för att du behöver kunna veta gränsvärdet för att lösa för gränsvärdet (i det här fallet för att veta vad derivatan av sin(x) är, för denna gränsvärdet ligger inuti derivatan av sin x när man använder sig av derivatans definition.). Däremot argumenteras det om att L'hopitals regel kan användas om man vill inte fokusera på det logiska. Det ger korrekt svar, fast logiken bakom det anses vara felaktigt. (jag tror det här är svaret i så fall). Källa: calculus - L'Hopital's rule and $\frac{\sin x}x$ - Mathematics Stack Exchange

Arup 1124
Postad: 11 jul 08:44

Finns det inget enklare sätt att lösa problemet avsett för ma 3 ?

Du är nästan framme i inlägg #5. Det är den metoden man skall använda i Ma3.

Arup 1124
Postad: 11 jul 11:09

Hur förenklar jag den sista raden 

limx(x2x2+3x+2)

?

Calle_K 2148
Postad: 11 jul 11:09

Dividera täljare och nämnare med x2

Arup 1124
Postad: 11 jul 11:11
Calle_K skrev:

Dividera täljare och nämnare med x2

Ok, så då får jag

limx(1+3x+x22)

Vad gör jag nu ?

Calle_K 2148
Postad: 11 jul 11:13

Det där stämmer tyvärr inte. Behåll kvoten, och dividera både täljare och nämnare var för sig med x2

Arup 1124
Postad: 11 jul 17:33

Calle K har du möjlighet att visa. Jag verkar helt glömmt bort detta

Calle_K 2148
Postad: 11 jul 17:34

Om du dividerar täljaren med x2 får du 1.

Vad får du om du dividerar nämnaren med x2?

Arup 1124
Postad: 11 jul 17:42

har du möjlighet att visa jag verkar ha tappat bort mig helt

Calle_K 2148
Postad: 11 jul 17:43

Vad är nämnaren i ditt bråk?

Arup 1124
Postad: 14 jul 13:31

Calle K jag förstår inte hur du menar med att dela var för sig ?

Arup 1124
Postad: 14 jul 13:42

Är det här rätt ?

shkan 215
Postad: 14 jul 14:18
Arup skrev:

Är det här rätt ?

Arup, ditt uttryck borde istället bli:

1/(1 + 3/x + 2/[x^2]), dvs  1(1 + 3x+2x2), för att du faktoriserar x^2 från både täljaren och nämnaren.

Vad händer nu då om du låter x gå mot oändligheten, vad händer?

Arup 1124
Postad: 14 jul 14:19

Tack, blir inte uttrycket så litet som möjligt ?

shkan 215
Postad: 14 jul 14:20 Redigerad: 14 jul 14:20
Arup skrev:

Tack, blir inte uttrycket så litet som möjligt ?

3/x går mot noll, och 2/x^2 går mot noll. Detta innebär att uttrycket blir 1/1, vilket motsvarar 1.

Arup 1124
Postad: 14 jul 14:20

hur då ?

shkan 215
Postad: 14 jul 14:21 Redigerad: 14 jul 14:21
Arup skrev:

hur då ?

Låt oss säga att x är ett jätte stort tal, kanske 1000000000000000000000000000 (kan vara mindre eller större än så). Vad händer om du sätter den där talen i 3/x, och 2/x^2?

Arup 1124
Postad: 14 jul 14:21

bråket blir väldigt  litet

shkan 215
Postad: 14 jul 14:22
Arup skrev:

bråket blir väldigt  litet

Exakt! Så om vi säger att x i 3/x och 2/x^2 går mot oändligheten, vilket värde tror du passar bäst i de hära uttrycken?

Arup 1124
Postad: 14 jul 14:24

1

shkan 215
Postad: 14 jul 14:29
Arup skrev:

1

det blir noll, för att x blir väldigt liten.

Arup 1124
Postad: 14 jul 14:30

Just det

Arup 1124
Postad: 14 jul 14:31

Så svaret är alltså att gränsvärdet existerar

shkan 215
Postad: 15 jul 10:47
Arup skrev:

Så svaret är alltså att gränsvärdet existerar

Ja

Svara Avbryt
Close